A is 5 times B, and B is 7 less than 2 times C. Which of the following statements describes the
relationship betweeen A & C?
(a) A is 7 less than 7 times c
(b) A is 7 less than 10 times C
(C) A is 14 less than 7 times C
(d) A is 35 less than 7 times C
(e) A is 35 less than 10 times C.

Answers

Answer 1

Answer:

C is the answer

Step-by-step explanation:

I'm sure it is


Related Questions

How would you describe the difference between the graphs of f(x)=2/3 x^3 and g(x)=2/3(-x)^3

Answers

The first equation or first graph with that equation has a positive slope and the other one has a negative slope

find the missing side of triangle​

Answers

Perpendicular=P=21Base=B=?Hypotenuse=H=29

Using Pythagoras Theorem

[tex]\\ \sf\longmapsto B^2=H^2-P^2[/tex]

[tex]\\ \sf\longmapsto B=\sqrt{H^2-P^2}[/tex]

[tex]\\ \sf\longmapsto B=\sqrt{29^2-21^2}[/tex]

[tex]\\ \sf\longmapsto B=\sqrt{841-441}[/tex]

[tex]\\ \sf\longmapsto B=\sqrt{400}[/tex]

[tex]\\ \sf\longmapsto B=20[/tex]

PLS HELP I'M SOO CONFUSED
Consider the arithmetic senes 336+ 332+328+....+4 Numeric Response 1 The number of terms in the series above is (Record your answer in the numerical-response section below) Your answer DCC Numeric Response 2. The sum of the series above is (Record your answer in the numerical-response section below) ​

Answers

The first term in the series is 336. The second term is obtained by subtracting 4 from the first term; subtract 4 from that to get the third term; and so on. Then the n-th term in the series is

336 - 4 (n - 1)

or

340 - 4n

The last term in this series is 4, so we solve for n :

340 - 4n = 4

336 = 4n

n = 336/4 = 84

The sum of the series is then

[tex]\displaystyle\sum_{n=1}^{84}(340-4n) = 340\sum_{n=1}^{84}1 - 4 \sum_{n=1}^{84}n[/tex]

Recall that

[tex]\displaystyle \sum_{n=1}^{N} 1 = N[/tex]

[tex]\displaystyle \sum_{n=1}^N n = \frac{N(N+1)}2[/tex]

Then the sum we want is

[tex]\displaystyle\sum_{n=1}^{84}(340-4n) = 340\times84 - 4\times\frac{84\times85}2 = \boxed{14,280}[/tex]

8/3=12/n solve for n

Answers

Answer:

[tex]n=\frac{9}{2}[/tex]

Step-by-step explanation:

[tex]\frac{8}{3} =\frac{12}{n}[/tex]

Cross multiply.

8n=12(3)

8n=36

[tex]n=\frac{9}{2}[/tex]

I hope this helps!

pls ❤ and give brainliest pls

Answer: n = 9/2

Step-by-step explanation:

8/3 = 12/n

Now doing cross multiplication

8/3 = 12/n

8(n) = 12(3)

8n = 36

n = 36/8

n = 9/2

Therefore value of n is 9/2

Must click thanks and mark brainliest



The figure shown to the right is an isosceles triangle, and
R is the midpoint of PS.
The fig
labeled
A. Explain when it is appropriate to use the statement PT TS.
P
R
S
B. Explain when it is appropriate to use the statement PT = TS.

Answers

Answer:

We know that an isosceles triangle has 2 of its sides being equal

With R, being the midpoint of PS, we can say that

PR=RS

Noting that, with R as midpoint, we can conclude that RT is a straight line which divides angles TPR and TSR into 2 right angle triangles

Step-by-step explanation:

therefore angle at P is 45°. Angle at S also 45°

Therefore PT = TS

This is because T is 45 degrees as well as P which is also 45 degrees

angle in triangle PTS is 180 degrees

R is 90 degrees, P is 45 degrees and the whole of T is also 45 degrees(which has been split into 2)

I need help please!!!!

Answers

a. 122.5 g

M(t) = 300(1/2)^(31/24) = 122.5



The perimeter of a rectangle is 56 feet and
its area is 192 square feet. What are the
dimensions of the rectangle?

Answers

Answer:

Step-by-step explanation:

P = 2(L + W)

Area = L*W

Area = 192

(L + W)*2 = 56

L+W = 28

L = 28 - W

W*(28 - W) = 192

28W - w^2 = 92

-w^2 + 28w - 192 = 0

w^2 - 28w + 192  = 0

This factors into

(w - 12)(w - 16) = 0

w - 12 = 0

w = 12

L = 28 - 12 = 16

see question in image

Answers

Answer:

b) 1/9

Step-by-step explanation:

Rolling two dice, there are 6*6 = 36 outcomes

The outcomes with the difference of 4:

1&5, 2&6, 6&2, 5&1 - total of 4

Required probability:

P = 4/36 = 1/9

Correct choice is b

please help. only need to do part b

Answers

Unless I’m reading it wrong, 20 students jumped 4 meters so 19 must have jumped less. As for more than 4.5, only 45 students jumped 4.5 meters, and only 60 students altogether, so the final 15 must have jumped more than 4.5.

What is the solution to this inequality?
-16x>-80
A. x < 5
O B. x>-5
O c. x<-5
O D. x>5

Answers

Answer:

A

Step-by-step explanation:

Divide both sides with -16. ALWAYS remember that if you divide any number with a negative number, this "< ≤ > ≥" symbols have to change to the opposite direction

You’re given two side lengths of 10 centimeters and 8 centimeters. The angle between the sides measures 40°. How many triangles can you construct using these measurements?

Answers

Answer:

1

Step-by-step explanation:

Once you have two sides and the included angle, there is only one triangle.

Answer: 1

Answer:

The answer is B. 1

Step-by-step explanation:

I hope I helped

Write in word form.
4. 572
5. 2.897
6. $325.10​

Answers

Answer:

heyyy thereee

4) 572 = five hundred seventy two

5) 2.897 = two and eight hundred ninety seven thousandths

6) three hundred twenty five and 10 hundredths  or three hundred twenty five and 1 tenth

glad if brainliest ig ?

When 3 liters of oil are removed from an upright cylindrical can, the level falls by 10cm. Find the radius of the can

Answers

Answer:

9.8 cm

Step-by-step explanation:

Volume of a cylinder can be gotten with this formula

V=πr2h

h = height of the oil.

We were told that 3 litres of oil was removed, then, there is a falling in the level of oil by 10 cm, hence the height of the oil is now

h° =( h) - (0.1)

Then the new volume can be calculated as

V°= V - (3×10^-3)

But

volume= (π r2h°)

Then substitute we have

πr2h - 0.003 = π r2 (h-0.1)

If we open the bracket we have

0.003 = (πr2 ×0.1)

πr2=0.003/0.1

Then r=√ (0.03/0.1π)

=9.8 cm

Note that 1L = 10-3 m3

Someone help pleaseee

Answers

Answer:

see explanation

Step-by-step explanation:

The area (A) of a rectangle is calculated as

A = length × breadth

   = (2 + [tex]\sqrt{2}[/tex] )(4-2[tex]\sqrt{2}[/tex] ) ← expand using FOIL

   = 8 - 4[tex]\sqrt{2}[/tex] + 4[tex]\sqrt{2}[/tex] - 4 ← collect like terms

   = 4 units²

--------------------------------------------------------

The opposite sides of a rectangle are congruent , so

perimeter = 2(4 - 2 [tex]\sqrt{2}[/tex]) + 2(2 + [tex]\sqrt{2}[/tex] ) ← distribute parenthesis

                 = 8 - 4[tex]\sqrt{2}[/tex] + 4 + 2[tex]\sqrt{2}[/tex] ← collect like terms

                 = 12 - 2[tex]\sqrt{2}[/tex] units

Solve for x 3x+3/x-4 = 3x+2/x+4

Answers

Answer:

here you go! with step by step so you can do it next time

Answer:

-7/8

Step-by-step explanation:

cross multiply first then expand the equations when you cross multiply it will be

(3x+3)(x+4)=(3x+2)(x-1)

3x(x+4)+3(x+4)=3x(x-1)+2(x-1)

3x²+12x+3x+12=3x²-3x+2x-2

3x²+15x+12=3x²-x-2

3x²-3x²+15x+x=-2-12

16x/16=-14/16

x=-14/16

simplified to

-7/8

I hope this helps

5. Solve the differential equation y" - 5y "+7y'-3y=0.​

Answers

The differential equation

y''' - 5y'' + 7y' - 3y = 0

has characteristic equation

r ³ - 5r ² + 7r - 3 = (r - 3) (r - 1)² = 0

with roots at r = 3 and r = 1 (with multiplicity 2), so that the characteristic solution to the DE is

y = C₁ exp(3x) + C₂ exp(x) + C₃ x exp(x)

HELP.... please??????????????

Answers

Answers:

Functions

y = -x+11y = 2x^2-6x+4y = -7

Not functions

x = 3x^2+y^2 = 81y^2 = -5x-12

=======================================================

Explanation:

A function is possible if and only if any given x input leads to exactly one y output.

For something like x^2+y^2 = 81, we can see that x = 0 leads to either y = 9 or y = -9. So this would not be a function. We would need x to pair with only y value to have it be a function.

We have the same thing going on with y^2 = -5x-12 as well.

For anything of the form x = k, where k is any real number, this is also not a function. We have one single input only and it leads to infinitely many outputs. So in a sense, this is even worse compared to the other examples.

-----------------

In summary, we have these three non-functions:

x = 3x^2+y^2 = 81y^2 = -5x-12

Everything else is a function. You can use the vertical line test as a visual way to check.

Question: "If y > 3, what is the value of n ?"

Answers

Answer:

y-3

Problem:

What is the remainder when the dividend is xy-3, the divisor is y, and the quotient is x-1. ?

Step-by-step explanation:

Dividend=quotient×divisor+remainder

So we have

xy-3=(x-1)×(y)+remainder

xy-3=(xy-y)+remainder *distributive property

Now we just need to figure out what polynomial goes in for the remainder so this will be a true identity.

We need to get rid of minus y so we need plus y in the remainder.

We also need minus 3 in the remainder.

So the remainder is y-3.

Let's try it out:

xy-3=(xy-y)+remainder

xy-3=(xy-y)+(y-3)

xy-3=xy-3 is what we wanted so we are done here.

What are the solutions to the system of equations?

{y=2x²−6x+3
{y=x−2

Answers

Answer:

x = 1, y = −1

x = 5/2, y = 1/2

Step-by-step explanation:

From the question given above, the following data were obtained:

y = 2x² − 6x + 3 ........ (1)

y = x − 2 ...... (2)

We can obtain the solutions to the equation as follow:

y = 2x² − 6x + 3 ........ (1)

y = x − 2 ...... (2)

Substitute the value of y in equation 2 into equation 1

y = 2x² − 6x + 3

y = x − 2

2x² − 6x + 3 = x − 2

Rearrange

2x² − 6x − x + 3 + 2 = 0

2x² − 7x + 5 = 0

Solve by factorization

Obtain the product of 2x² and 5. The result is 10x².

Find two factors of 10x² such that their sum will result to −7x.

The factors are −2x and −5x.

Replace −7x in the equation above with −2x and −5x as shown below:

2x² − 2x − 5x + 5 = 0

2x(x − 1) − 5(x − 1) = 0

(x − 1)(2x − 5) = 0

x − 1 = 0 or 2x − 5 = 0

x = 1 or 2x = 5

x = 1 or x = 5/2

Substitute the value of x into equation 2 to obtain y

y = x − 2

x = 1

y = 1 − 2

y = −1

x = 5/2

y = x − 2

y = 5/2 − 2

y = (5 − 4)/2

y = 1/2

SUMMARY:

x = 1, y = −1

x = 5/2, y = 1/2

15) The sum of one-fourth, one-ninth and one- third of a number is 25. Find the number.​

Answers

Answer:

Step-by-step explanation:

(1/4 + 1/9 + 1/3)x = 25.

x = 36

Answer:

Let the number be x

(1/4 + 1/9 + 1/3) * x = 25

25/36 * x = 25

25x = 25*36

x = 25*36/25

x = 36

Step-by-step explanation:



Which composite function can be used to find the
force of the object based on its volume?

The density of titanium is 4.5 g/cm3. A titanium object
is accelerating at a rate of 800 cm/s2. The mass of
the object can be modeled by the function m(v) =
4.5v, where v is the volume in cubic centimeters.
Additionally, the force of the object can be found
using the function F(m) = 800m.

A. F(m(v)) = 177.8V
B. F(m(v)) = 795.5v
C. F(m(v)) = 804.5v
D. F(m(V)) = 3,600V

Answers

Given:

The mass function is:

[tex]m(v)=4.5v[/tex]

where v is the volume in cubic centimeters.

The force function is:

[tex]F(m)=800m[/tex]

To find:

The composite function can be used to find the force of the object based on its volume.

Solution:

The composite function can be used to find the force of the object based on its volume is:

[tex]F(m(v))=F(4.5v)[/tex]              [tex][\because m(v)=4.5v][/tex]

[tex]F(m(v))=800(4.5v)[/tex]              [tex][\because F(m)=800m][/tex]

[tex]F(m(v))=3600v[/tex]

Therefore, the correct option is D.

Answer: F(m(v)) = 3,600v

Step-by-step explanation:DDDD

Sara brought a car for 16,586 and paid 1,038 for tax, title, and registration.Which equation shows about how much Sara paid, to the nearest hundred?

Answers

Answer:

P+T+t+R=TA

Step-by-step explanation:

this should help

Answer:

17,600

Step-by-step explanation:

16,586 + 1,038 = 17,624 ≈ 17,600

find the missing side​

Answers

Answer:

x ≈ 13.7

Step-by-step explanation:

Using the cosine ratio in the right triangle

cos70° = [tex]\frac{adjacent}{hypotenuse}[/tex] = [tex]\frac{x}{40}[/tex] ( multiply both sides by 40 )

40 × cos70° = x , then

x ≈ 13.7 ( to the nearest tenth )

simplify 2 root 3 multiply by root 7​

Answers

4x4x7
12x7
= 84 maybe this????????

Answer:

[tex]2 \sqrt{3} \times \sqrt{7} = 2 \sqrt{21} = 2 \times 4.58 = 9.16[/tex]

I hope I helped you^_^

In circle O, and are diameters. The measure of arc AB is 55° and the measure of arc CD is 25°.

Circle O is shown. Line segments A D and B E are diameters. Line segments O F and O C are radii.

What is the measure of Arc E A C?

Answers

Your Answer Is in this attachment.

Answer:

D-212 Degrees

Step-by-step explanation

i got it right on edge

which value of g makes 26=7(g-9)+12 a true statment

Answers

Answer:

11

Step-by-step explanation:

26=7(g-9)+12

14=7(g-9)

2=g-9

g=11

what should be added to 4.289 to get 11​

Answers

[tex]\huge\text{Hey there!}[/tex]

[tex]\large\text{We do not know the unknown number just yet so we will label it}\\\large\text{as the variable of \boxed{\bf n}}\large\text{ until we find the result of the unknown}\\\large\text{number}[/tex]

[tex]\large\text{So, your equation is now: \underline{\underline{n + 4.289 = 11}} or \underline{\underline{4.289 + n = 11}}}[/tex]

[tex]\large\textsf{n + 4.289 = 11}\\\large\text{SUBTRACT \underline{4.289} to BOTH SIDES}\\\large\text{n + 4.289 - 4.289 = 11 - 4.289}\\\large\text{CANCEL out: 4.289 - 4.289 because that gives you 0}\\\large\text{KEEP: 11 - 4.289 because that helps you get the n-value}\\\large\text{SIMPLIFY ABOVE AND YOU HAVE YOUR RESULT}\\\large\text{n = \bf 6.711}\\\\\boxed{\boxed{\huge\text{Therefore, your answer is: \bf 6.711}}}\huge\checkmark[/tex]

[tex]\huge\text{Good luck on your assignment \& enjoy your day!}[/tex]

~[tex]\frak{Amphitrite1040:)}[/tex]

Let the number which should be added is x

ATQ

[tex]\\ \sf\longmapsto x+4.289=11[/tex]

Take 4.289 to right

[tex]\\ \sf\longmapsto x=11-4.289[/tex]

[tex]\\ \sf\longmapsto x=6.711[/tex]

6.711 should be added to 4.289 to get 11

What is the domain of the function RX) = x2 + 3x + 5?
OA
all whole numbers
OB. all positive real numbers
Ос.
all Integers
OD. all real numbers

Answers

Answer:

all real numbers

Step-by-step explanation:

The domain is the numbers that the input, x, can take

We can put in any real number for x since there are no restrictions on the input

Answer:

option D is domain

Step-by-step explanation:

because no change is happen in range by keeping any real number

helpppppppp will mark brainlest​

Answers

Answer:

-8

Step-by-step explanation:

what do you think, when you look at the examples given in the problem definition ?

don't you see the pattern, that f(x) = x+2 ?

f(1) = 1+2 = 3

f(2) = 2+2 = 4

f(3) = 3+2 = 5

so, if we follow this assumption, then

f(-10) = -10 + 2 = -8

[(-33) + 46 + 68] - |15 - 61|

Answers

Answer:

127

Step-by-step explanation:

[(-33) + 46 + 68] - (15 - 61)

(114-33)-(-46)

81+46

127

Other Questions
find the solution to the system of equations. y= -7x + 3y= -x - 3 Please help! Thank you. I need An answer ASAP!? What were the three large, flightless birds with similar features found on different continents, Australia, Africa, and South America? The sum of fourconsecutive odd number is 8o. Find the number Government works to maintain a competitive business environment by preventing __________.A.too many new companies going into business all at onceB.one business from unfairly taking over a marketC.competition from outside the USD.prices from becoming too high If 24n- + 3 = 7 What is the value of n Which of the following statements about software licensing are true? Select 3 options.A. Both proprietary and open-source software can be modified and redistributed. B.Free software does not come with a license. C.Open-source software allows developers to create new software more quickly and inexpensively. D. Proprietary software licenses are more restrictive than open-source software licenses.E. An end-user license agreement protects both the owner of the software and the purchaser. Write a memoir of 250 to 300 words on the following topic :Any historical incident that impacted your life dog wants to get better at volleyball. In order to do this he has created a list of skills she will need to improve. Which skill should get the most of his attention in order for him to excel at volleyball Multiply the polynomial by distribution. Show your work and explain the steps you used to solve.- 8x(x^2 8x + 3) Where does change management play a major role in transforming a clientbusiness into a data-driven, intelligent enterprise? Question 3:Which of these is NOT a way you can be exposed to chemicals?Click on the correct answer below, then click the Submit button.A) Splashing in your eyesB) Holding chemicals while wearing vinyl glovesC) Breathing in dust or vaporD) Absorption through your skinE) Swallowing themSubmit Why are digital computer known as general purpose computer why do solvents particles flow into the cell when the initial volume is below 50% Choose the vocabulary word that BEST completes the sentence.The food writer ate at the restaurant on three nights to get an accurate picture of itsquality.hoardingsubduedmythologicalsuccessive CAN SOMEONE PLEASE HELP WRITE ME A POEM BASED OFF OF IDENTITY!!poem must include : - a unique title - my love for swimming in the ocean and how it makes me feel calm - show how the ocean makes me who i am- poetic devices such as figurative language -how my thoughts disappear when im at the ocean these things technically make up identity so yeah help, im bad at making poems !!! If position vector r=bti + ctj, where c are positive constants, when does the velocity vector make an angle of 45 with the x and y axes? Which law prohibits companies receiving federalfinancial assistance from treating an individualdifferently when determining eligibility?A. Title VI of the Civil Rights Act of 1964B. The National CLAS StandardsC. The Dodd-Frank Act D. HIPAA Need helppppp asappppppp